8
$\begingroup$

Let $q$ be a prime power. It is well known that all Singer subgroups (subgroups of order $q^n-1$) in $GL(n,q)$ are conjugate. My question is: If $H$ is a cyclic subgroup of order $m$ in $GL(n,q)$, $m\mid (q^n-1)$ and $m$ is quite large (for example, $m= (q^n-1)/2$ for $q$ odd), is it necessary that $H$ must be contained in some Singer subgroup of $GL(n,q)$?

$\endgroup$

1 Answer 1

11
$\begingroup$

Yes. Put the generator in rational canonical form. Because its order is prime to $q$, it is semisimple, so this puts it in block diagonal form where each block is a companion matrix of an irreducible.

It is contained in a Singer subgroup if and only if it has a single $n \times n$ block.

Suppose it is not in a Singer subgroup. Then the order of the matrix is the lcm of the order of each block. Since each $k \times k$ block is in a Singer subgroup, its order divides $q^{k}-1$. The lcm of all these copies of $q^{k}-1$ is at most their product divided by $q-1$, since $q-1$ divides them all, and there are at least two blocks. Since the $k$s add up to $n$, the product is less than $q^n-1$. So if it is not Singer, the order must be less than $q^n-1/ (q-1)$. So as long as $m \geq (q^n-1)/(q-1)$, there is a contradiction, and it must be Singer.

$\endgroup$
2
  • $\begingroup$ Is the statement "It is contained in a Singer subgroup if and only if it has a single $n \times n$ block." really correct? A proper subgroup of the Singer subgroup need not have a single $n \times n$ block, right? (for example, consider the trivial subgroup) $\endgroup$ Mar 25, 2018 at 11:56
  • 1
    $\begingroup$ @scalar You're right, I think the correct criterion is it is contained in a Singer if all the blocks are equal, but in any case the argument uses only the "if" direction. $\endgroup$
    – Will Sawin
    Mar 25, 2018 at 13:11

Your Answer

By clicking “Post Your Answer”, you agree to our terms of service and acknowledge you have read our privacy policy.

Not the answer you're looking for? Browse other questions tagged or ask your own question.